Find an equation in slope-intercept form of the line that has slope –7 and passes through point A(8,-4)
a.) y = 52x + 7
b.) y = 7x + 52
c.) y = -7x - 52
d.) y = -7x + 52

Answers

Answer 1

Answer:

y = -7x + 52

Step-by-step explanation:

y = -7x - b

-4 = -7(8)  + b

-4 = -56 + b

52


Related Questions

solve for the measure of a

Answers

Answer:

Step-by-step explanation:

The supplement of the exterior angle is 180 - 75 = 105

In a cyclic quadrilateral (a four sides figure where all the vertices touch the circumference of a circle), the opposite angles are supplementary. Therefore <a = 180 - 105 = 75

But I think you already knew that.

In the figure below j ll h find the values of x and y

Answers

Answer:

26

Step-by-step explanation:

Given: Lines J and H are parallel lines. O intersects both lines equally.

Since two angles equal 180, that will be what  (4y + 12) and 64 will be equal to.

(4y + 12) + 64 = 180

76 + 4y = 180

-76          -76

-------------------

    4y = 104

      y = 26

x and y is both 26 because they are vertical angles.

Hope this helped.

what must be added to a + b to get a​

Answers

Answer:

we have to add (-b) to get (a)

Step-by-step explanation:

a+b+(-b)=a

a+b-b=a

a=a

Hence verified.

Hope this helps you. Have a nice day^_^

(6 1/4)^4

Answer fast or i will report you

Answers

Answer:6

Step-by-step explanation: The rules of exponential says (a^x)^y=a^xy.

Therefore you will multiply 1/4 with 4 to get an exponent of 1. So the answer is 6^1 which is also written as 6

Help fast!
Describe at least two ways to find or
estimate the year the population of the town
will be 40 thousand. (You don't have to
actually find the value.)

Answers

Plug in 40,000 for p(n), then use algebra. First step divide by 18. Then you have 2222.222222=e^0.035n.
Take the natural logarithm (Ln) on your calculator of both sides:
Ln(2222.2222)=Ln(e) 0.035n
Ln2222.2222 just gives you a number and Ln(e) cancel each other so you have:
Ln(2222.222222)= 0.035n, divide both sides by 0.035 and you will find “n”(the number of years after 2000).

The second method is trial and error, keep plugging in numbers for n starting at “1” and see what number gets you closest to 40,000 when you calculate the equation

In XYZ, what is the cosine ratio of X?

Answers

Answer:

c) 12/15 = 4/5

Step-by-step explanation:

imagine we mirror the triangle up, so that Z is on top.

then you can clearly see that 6 is cos(X) times r (and r is then 7.5).

XY is sin(X)×7.5

and again, 7.5 is r (the line making the X angle).

so, the cosine ratio of X is

6 = cos(X)×7.5

cos(X) = 6/7.5 or then 12/15. or simplified 4/5.

yea the person on top is right, bc I also got that correct on my test LMFOAOA

Vanessa and her friends are watching three movies consecutively. The first movie is 2 hours and 17 minutes long. The second movie is 84 minutes long, and the last movie is 99 minutes long. How much time will they spend watching the movies?

Answers

Answer:

320 minutes (5 hours and 20 minutes).

Step-by-step explanation:

2 hours and 17 minutes = 137 minutes

137 + 84 + 99 = 320

Therefore, they will spend 320 minutes (5 hours and 20 minutes) watching movies.

Slope -1/4, passes through (12,-4)​

Answers

Answer:

y = - [tex]\frac{1}{4}[/tex] x - 1

Step-by-step explanation:

Assuming you require the equation of the line

The equation of a line in slope- intercept form is

y = mx + c ( m is the slope and c the y- intercept )

Here m = - [tex]\frac{1}{4}[/tex] , then

y = - [tex]\frac{1}{4}[/tex] x + c ← is the partial equation

To find c substitute (12, - 4) into the partial equation

- 4 = - 3 + c ⇒ c = - 4 + 3 = - 1

y = - [tex]\frac{1}{4}[/tex] x - 1 ← equation of line

On a coordinate plane, a curved line labeled f of x with a minimum value of (1.9, negative 5.7) and a maximum value of (0, 2), crosses the x-axis at (negative 0.7, 0), (0.76, 0), and (2.5, 0), and crosses the y-axis at (0, 2).
Which statement is true about the graphed function?

F(x) < 0 over the intervals (-∞, -0.7) and (0.76, 2.5).
F(x) > 0 over the intervals (-∞, -0.7) and (0.76, 2.5).
F(x) < 0 over the intervals (-0.7, 0.76) and (2.5, ∞).
F(x) > 0 over the intervals (-0.7, 0.76) and (0.76, ∞

Answers

Answer:

F(x) < 0 over the intervals (-∞, -0.7) and (0.76, 2.5)

Step-by-step explanation:

The minimum value of the curve = (1.9, -5.7),

The maximum value = (0, 2)

The point the function crosses the x-axis (the x-intercept) = (-0.7, 0), (0.76, 0), and (2.5, 0)

The point the function crosses the y-axis (the y-intercept) = (0, 2)

The given points can be plotted using MS Excel, from which we have;

F(x) is less than 0 over the interval from x = -∞, to x = -0.7, and the interval from x = 0.76 to x = 2.5

The correct option is therefore, F(x) < 0 over the intervals (-∞, -0.7) and (0.76, 2.5)

Answer:

A. F(x) < 0 over the intervals (-∞, -0.7) and (0.76, 2.5).

Step-by-step explanation:

RS=7y+4, ST=3y+6, and RT=90

Answers

Answer:

If it is a straight line then ;

RT= RS+ST

90=(7y+4) + (3y+6)

90 = 10y + 10

10y= 90 – 10

10y = 80

y= 80 / 10

y =8

ST = 3y+6= 3(8)+6= 24 +6 = 30

RS = 7y +4 = 7(8) + 4 = 56 +4 = 60

I hope I helped you^_^

How to find interquartilte range

Answers

Answer:  Choice C) 8.5

============================================================

Explanation:

Each x represents a data point location.

So, for example, having an x over 60 means 60 is part of the set.

The set of values we're working with is

{59,60,61,63,63,64,66,68,70,71,71,73}

The repeated values are due to the fact we have a stack of two 'x' markers, and they occur at 63 and 71.

To find the IQR (interquartile range), we'll first need to find the median of this set. That's the middle most value.

Count out the number of values to find that there are n = 12 values.

The list splits into two halves that are n/2 = 12/2 = 6 items each

Between slots 6 and 7 is where the median is located.

The value in slot 6 is 64 and the value in slot 7 is 66. Average those two items to get (64+66)/2 = 65

The median is 65

---------------------------------

Next, we'll form two groups L and U such that

L = set of items lower than the median

U = set of items larger than the median

Because n is even, we simply just break the original set into two equal groups (6 items each)

L = {59,60,61,63,63,64}

U = {66,68,70,71,71,73}

The values of Q1 and Q3 represent the medians of L and U in that order.

The median of set L is (61+63)/2 = 62, so Q1 = 62

The median of set U is (70+71)/2 = 70.5, which is Q3

-----------------------------------

To summarize everything so far, we have found

Q1 = 62Q3 = 70.5

Subtract those items to get the IQR

IQR = Q3 - Q1

IQR = 70.5 - 62

IQR = 8.5 which points us to choice C as the final answer.

Alec pulled a couch 3 meters, using a force of 400 N. The couch weighed 200 N. How do you calculate the work done by Alec?

A . Add 400 to 200

B . Divide 400 by 3

C . Multiply 200 by 3

D . Multiply 400 by 3

Answers

Answer:

D

Step-by-step explanation:

It is because work is done when a force cause an object to move in the direction of the applied force.

so work is equal to force × distance

arshad's father bought x sweets .(x-4)were eaten by children and 20 were left.how many sweets did his father bring

Answers

Answer:

24

Step-by-step explanation:

20+4

simple

x-4=20

x=20+4

x=24

mark me as brainliest

Answer:

24 sweets

Step-by-step explanation:

Remaining sweets = 20

x - 4 = 20

Add 4 to both sides.

x = 20 +4

x = 24

the tub started with gallons of water

Answers

Answer:

huh?

Step-by-step explanation:

Please help me solve this problem

Answers

Answer:

-4

they wanted you to compute using x as 3

-2*3 + 2 = -4

Step-by-step explanation:

pls help, and explain. I will give brainliest

Answers

Answer:

Nicole should take 13 1/8 cups of snack mix.

Step-by-step explanation:

If a serving size is 7/8 and Nicole wants to take 15 serving sizes (15 7/8's), then we must multiply 7/8 by 15:

15 × 7/8

Write 15 over 1 (15 = 15/1) to make the calculations easier:

15/1 × 7/8

Multiply the numerators and denominators separately:

15 × 7 / 1 × 8

105 / 8

Instructions don't say you have to do this but I will convert this improper fraction into a mixed number:

105/8 = 13 1/8

Please someone tell me the answer of these questions

Answers

Answer:

VERTICALLY OPP ANGLES

Step-by-step explanation:

The perimeter of a rectangle is 18cm . if the length is (x+2), find it's width.​

Answers

Answer:

W = 7 - x

Step-by-step explanation:

The perimeter is P= 2L + 2×W , where L is the length and W is the width.

If L = (x+2) , replacing L with the expression x+2 we have

P= 2×(X+2) + 2W ⇔ 18 = 2x + 4 + 2W ⇔ 2W =18 - 2x - 4 ⇔ 2W = 14 - 2x

⇔ W = 7 - x

Match function with its corresponding graph

Answers

Answer:

Step-by-step explanation:

We can see that there are roots at (-2,0) and (-1,0)

also, the root at (-2,0) should bounce right off

and the root at (-1,0) should go through

With all that being said it has to be B

At a particular restaurant, each slider has 225 calories and each chicken wing has 70 calories. A combination meal with sliders and chicken wings has a total of 7 sliders and chicken wings altogether and contains 1110 calories. Write a system of equations that could be used to determine the number of sliders in the combination meal and the number of chicken wings in the combination meal. Define the variables that you use to write the system.

Answers

Answer:

X+y=7

Step-by-step explanation:

i  remember  doing something like this but mines had the word onion rings .

9. Find the remainder when the polynomial: p(x) = x⁴ + 2x³- 3x² + x - 1 is divided by (x - 2)
pls it's urgent

Answers

Answer:

answer is 21..............

Explanation:

p(x) = x⁴ + 2x³- 3x² + x - 1

Factor of p(x)

x-2=0

x=2

Then by using synthetic division

y=8200(0.96)^x growth or decay find

Answers

Answer:

This would be a .04 or 4% decay.....

for every "time unit" (x in this case) you will be multiplying

the amount by .96 ... in other words if you started with one dollar

the results would be 96 cents... after two "time" steps you would have

only 92 cents (.96 *.96)

Step-by-step explanation:

which inequality is represented on the number line shown?

Answers

Answer: A x> -2

Step-by-step explanation:

Given an arithmetic progression 17,13,9,..... find the number of terms required so that its sum is - 33 .

Answers

Answer:

11 terms.

Step-by-step explanation:

We are given the arithmetic sequence:

17, 13, 9, ...

And we want to find the number of terms required such that the sum is -33.

Recall that the sum of an arithmetic series is given by:

[tex]\displaystyle S = \frac{k}{2}\left( a + x_k\right)[/tex]

Where k is the number of terms, a is the first term, and x_k is the last term.

The desired sum is -33. The first term is 17 as well. Thus:

[tex]\displaystyle (-33) = \frac{k}{2} \left( (17) +x_k\right)[/tex]

Simplify:

[tex]-66 = k(17 + x_k)[/tex]

We can write a direct formula to find the last term x_k. The direct formula of an arithmetic sequence has the form:

[tex]x_ n = a + d(n-1)[/tex]

Where a is the initial term and d is the common difference.

The initial term is 17 and the common difference is -4. Hence:

[tex]\displaystyle x_n = 17 - 4(n-1)[/tex]

Then the last term is given by:

[tex]x_k = 17 - 4(k-1)[/tex]

Substitute:

[tex]\displaystyle -66 = k\left( 17 + \left( 17 - 4(k-1)\right)\right)[/tex]

Solve for k:

[tex]\displaystyle \begin{aligned} -66 &= k(17 + (17 - 4k + 4)) \\ -66 &= k(38 -4k) \\ -66 &= -4k^2 + 38k \\ 4k^2 -38k -66 &= 0 \\ 2k^2 - 19k -33 &= 0 \\ (k-11)(2k+3) &= 0 \\ k-11&= 0 \text{ or } 2k+3 = 0 \\ \\ k &= 11 \text{ or } k = -\frac{3}{2}\end{aligned}[/tex]

Since we cannot have a negative amount of terms, we can ignore the second solution.

Therefore, the given sequence must have 11 terms such that it sums to -33.

Answer:

Here is 2 methods

Step-by-step explanation:

1) we use excel to find n=11 for lasy students

2) mathematical method

[tex]u_1=17\\u_2=13=17+(2-1)*(-4)\\u_3=9=17+(3-1)*(-4)\\\\\\\boxed{u_n=17+(n-1)*(-4)}\\\\\\\displaystyle s_n=\sum_{i=1}^nu_i\\=\sum_{i=1}^n(17+(i-1)*(-4))\\\\\\=(\sum_{i=1}^n 17) + (-4)*\sum_{i=1}^n (i) +4*\sum_{i=1}^n (1)\\\\\\=17*n+4*n-4*\frac{n*(n+1)}{2} \\\\\\=21n-2n^2-2n\\\\\\=-2n^2+19n\\\\=-33\\\\\\\Longrightarrow\ 2n^2-19n-33=0[/tex]

[tex]\Delta=19^2+4*2*33=625=25^2\\\\n=\dfrac{19-25}{4} =-1.5\ (excluded)\ or\ n=\dfrac{19+25}{4}=11\\\\[/tex]

Triangle Q R S is shown. Line R Q extends through point P. Angle Q S R is 35 degrees. Angle S R Q is 58 degrees. Exterior angle S Q P is x degrees. What is the value of x?

Answers

The triangle is missing and so i have attached it.

Answer:

x = 93°

Step-by-step explanation:

From the triangle attached, we can say that;

<SQP + <SQR = 180°

This is because sum of angles on a straight line equals 180°.

Secondly, we know that sum of angles in a triangle also equals 180°.

Thus;

<SQR + <QSR + <SRQ = 180

From the attached triangle, we see that;

<QSR = 35°

<SRQ = 58°

Thus;

<SQR + 35° + 58° = 180°

<SQR + 93° = 180°

<SQR = 180° - 93°

<SQR = 87°

From earlier on, we saw that;

<SQP + <SQR = 180°

Plugging in <SQR = 87°, we have;

<SQP + 87° = 180°

<SQP = 180° - 87°

<SQP = 93°

We are told in the question that <SQP is denoted by x.

Thus;

x = 93°

Answer:

The value of x is answer D: 93

A person walks on average 4000 steps per day. If one step is about 2 feet long, how much would the average person walk per week? HELP

Answers

Answer:

56000 ft

Step-by-step explanation:

4000 steps a day.

7 days in a week.

2 ft per step

so, we calculate how many steps in a week

4000 × 7 = 28000

and then we calculate the distance by saying each of these steps is 2 ft

so,

28000 × 2 = 56000 ft

as a little extra thought :

there are 5280 ft in a mile.

so, the person walks

56000 / 5280 miles = 10.61 miles

in a week.

X+3y=2 and y=2x+3
Please explain using substitution method.

Answers

- X + 3Y = 2  (*)

⇔X = 2 - 3Y  (1)

- Y = 2X + 3   (2)

(1),(2)⇒ Y = 2(2 - 3Y) +3

       ⇔ Y = 4 - 6Y + 3

       ⇔ Y = 1  (**)

(*),(**)⇒ X + 3×1 =2

       ⇔ X = -1

the length of a rectangular box is 8cm. If its diagonal is 10cm. Find its width​

Answers

Answer:

Step-by-step explanation:

The diagonal of this rectangular box serves as the hypotenuse of the 2 right triangles that exist within this rectangle. The length is one leg, the hypotenuse is...well, the hypotenuse, so we need to use Pythagorean's Theorem to find the missing leg.

[tex]10^2=8^2+x^2[/tex] and

[tex]100-64=x^2[/tex] and

[tex]x^2=36[/tex] so

x = 6. The width is 6.

1.Evaluate

a.(243/32)^-0.4​

Answers

Answer:

4/9

Step-by-step explanation:

negative exponent means 1/...

so, this is

(32/243)^(4/10) = (32/243)^(2/5)

that means to the power of 2 and then pulling the 5th root.

so, let's pull the 5th root first, and then we square

(32/243)^(2/5) = (2⁵/3⁵)^(2/5) = (2/3)^2 = 4/9

If n(a)=40,n(b)=60 and n(a∪b)=80.find the value of (a∩b.

Answers

Answer:

20

Step-by-step explanation:

we know,

n(aUb)=n(a)+n(b)-n(a∩b)

so,

80=40+60-n(a∩b)

or, 80-(40+60)=-n(a∩b)

or, -n(a∩b)=80-100

or, -n(a∩b)=-20

or, n(a∩b)=20

Other Questions
BRAINLIESTHOW CAN POSTULATES AND THEOREMS RELATING TO SIMILAR AND CONGRUENT ANGLES BE USED TO WRITE A PROOF? Janice has been invited to appear on a home improvement show for the remodel of her summerhouse in Maine. Janice asks Mary to wallpaper her house in anticipation of the home improvement, and requests expensive custom wallpaper and a very intricate design application, for which the wallpaper would cost $5000, plus labor. Mary, excited for a very large job for her solo business, orders the intricate wallpaper and blocks off her calendar for the amount of time it will take to complete the job. After the paper has been ordered, Mary asks some friends to be available to complete the job in time for the show. Janice is informed that she will not be on the show and notifies Mary that she will not need the wallpaper. a. Does Mary have a case for re-imbursement?b. Under what legal theory might she prevail and what are her damages, if any?c. What ethical theories might be applicable? if x-5 varies directly as y and x=30 when y= 2, find the value of x when y=8. The distance from the plane to the building __ meters An observer measures a 100 Hz Doppler shift as an ambulance goes by. At rest, the frequency of the ambulance's siren is 2,000 Hz. What is the speed of the ambulance? how has our Constitution managed to maintain propertional representation on all areas of the sate?explane The New Orleans-born composer who incorporated African-Caribbean rhythms and Creole melodies in his compositions was A. Lowell MasonB. Louis Moreau GottschalkC. Charles Ives D. Scott Joplin A 771.0-kg copper bar is melted in a smelter. The initial temperature of the copper is 300.0 K. How much heat must the smelter produce to completely melt the copper bar? For solid copper, the specific heat is 386 J/kg K, the heat of fusion is 205 kJ/kg, and the melting point is 1357 K. Please give these answers hindi A line is perpendicular to the line y = 4x - 3 and has x-intercept (2,0). Which of the following is an equation of the line? i want the answer in tamil now It says 5.60 on the label. You've ... me 6.50.a. takenb. deductedc. reducedd. charged In January Mr.Tan's students read 20 books. In February, they read 15 books. What is the percent increase or decrease in the number of books the read? which is odd one out 4:10 12:25 20:50 8:20 Square root 1.000441 find the solution to the system of equations. y= -7x + 3y= -x - 3 Please help! Thank you. The sum of fourconsecutive odd number is 8o. Find the number Write a memoir of 250 to 300 words on the following topic :Any historical incident that impacted your life Choose the vocabulary word that BEST completes the sentence.The food writer ate at the restaurant on three nights to get an accurate picture of itsquality.hoardingsubduedmythologicalsuccessive